Fórum de Matemática
DÚVIDAS? Nós respondemos!

Um Fórum em Português dedicado à Matemática
Data/Hora: 28 mar 2024, 12:18

Os Horários são TMG [ DST ]


A pesquisa obteve 673 resultados
Procurar estes resultados:

Autor Mensagem

 Fórum: Números complexos   Pergunta: Relação entre seno e cosseno hiperbólico envolvendo os números complexos

Enviado: 07 dez 2014, 17:47 

Respostas: 1
Exibições: 1404


\text{cosh}(z) \text{cosh}(x+yi) \text{cosh}(x+yi)=\text{cosh}(x) * \text{cosh}(yi)+\text{senh}(x)*\text{senh}(yi) mas sabemos que : \text{cos}(iz)=\text{cosh}(z) , se z=yi teremos : \text{cos}(-y)=\text{cosh}(yi...

 Fórum: Limites de funções   Pergunta: Limite sucessões - Cálculo informático 1º Ano

Enviado: 10 nov 2014, 15:41 

Respostas: 5
Exibições: 4152


Olá :D


Veja esta videoaula por volta dos 35:00 é comentado a ordem de crescimento de funções.

 Fórum: Cálculo de integrais múltiplos   Pergunta: Cálculo de volume por integral tripla

Enviado: 10 nov 2014, 05:14 

Respostas: 1
Exibições: 1794


Veja este tópico : viewtopic.php?f=10&t=7034

 Fórum: Limites de funções   Pergunta: Limite sucessões - Cálculo informático 1º Ano

Enviado: 10 nov 2014, 05:10 

Respostas: 5
Exibições: 4152


Olá :D Perceba que temos algo do tipo : \lim_{n \to +\infty} \; \frac{f(n)}{g(n)} Então podemos usar a ordem de crescimento para determinar o valor. Perceba que f(n)=n^{10}+\frac{n}{5^{n+1}}+100 tem ordem O(f(n))=n^{k} , já g(n)=2*4^{n}+n^3+n tem ordem...

 Fórum: Primitivas e Integrais   Pergunta: integrais envolvendo funções trigonometricas e limites impróprios

Enviado: 08 nov 2014, 21:29 

Respostas: 3
Exibições: 2260


este exercício saiu no teste do ano passado... então em situação de prova a minha resolução ficava assim e depois punha divergente?´ obrigada Vc argumenta que a função seno é periódica então quando "b" tender a mais infinito o limite é um valor indefinido entre -1 e 1 , logo o limite não ...

 Fórum: Primitivas e Integrais   Pergunta: integrais envolvendo funções trigonometricas e limites impróprios

Enviado: 08 nov 2014, 17:33 

Respostas: 3
Exibições: 2260


Esta integral não converge.


https://www.wolframalpha.com/input/?i=i ... +to+%2Binf

 Fórum: Análise de Funções   Pergunta: Mostrar que se f é derivavel no ponto a, entao....

Enviado: 24 Oct 2014, 03:43 

Respostas: 1
Exibições: 1363


Boa tarde, Seja f : I -> R, definida num intervalo do qual a é ponto interior. Mostre que se f é derivável no ponto a, então lim_{h\rightarrow \0}\frac{f(a+h)-f(a-h)}{2h}=f'(a) \lim_{h \to 0} \; \frac{f(a+h)-f(a-h)}{2h} \lim_{h \to 0} \; \frac{f(a+h&#...

 Fórum: Cálculo de integrais múltiplos   Pergunta: integral tripla por coordenada cilíndrica

Enviado: 07 Oct 2014, 14:13 

Respostas: 8
Exibições: 5692


Tens razão Sobolev , então fica três formas de resolver (duas no link que enviei) e uma sua.


Obrigado :) :) :)

 Fórum: Limites de funções   Pergunta: cálculo acerca do limite fundamental trigonométrico

Enviado: 07 Oct 2014, 00:15 

Respostas: 2
Exibições: 1671


Olá :D Este limite não é pra ser feito usando o limite fundamental, e sim o teorema da função limitada que diz: q se no produto de duas funções ,uma tem o limite igual a zero e a outra função é limitada, o limite do produto é zero. \lim_{x \to 0} \; x \; \sin \left( \frac{1}{x} \right) \lim_...

 Fórum: Cálculo de integrais múltiplos   Pergunta: integral tripla por coordenada cilíndrica

Enviado: 07 Oct 2014, 00:02 

Respostas: 8
Exibições: 5692


Olá :D Sobolev a expressão caiu numa função eliptica logo não é integravél, o que se pode fazer para resolver nesses casos é aproximar usando a série binomial.Ou partir para outra modo de resolver sem a utilização de coordenadas cilindricas.Vide o exercício resolvido abaixo: http://forumdematematica...
Ordenar por:  
Página 3 de 68 [ A pesquisa obteve 673 resultados ]


Os Horários são TMG [ DST ]


Ir para:  
cron